Đến nội dung

Hình ảnh

ÔN THI ĐẠI HỌC 2012


  • Please log in to reply
Chủ đề này có 95 trả lời

#61
yeutoan11

yeutoan11

    Sĩ quan

  • Thành viên
  • 307 Bài viết

Bài toán 36.
Ch0 các số thực không âm $a,b,c$ thỏa mãn $ab+bc+ca=1$.Tìm giá trị nhỏ nhất của biểu thức:
$$D=\frac{1}{a+b}+\frac{1}{b+c}+\frac{1}{c+a}-\frac{1}{a+b+c}$$


36)
Thay 2 bộ : $(1,1,0)$ và $(\frac{1}{\sqrt{3}};\frac{1}{\sqrt{3}};\frac{1}{\sqrt{3}})$
Ta thấy bộ $(1,1,0)$ thì $D=2$ còn bộ $(\frac{1}{\sqrt{3}};\frac{1}{\sqrt{3}};\frac{1}{\sqrt{3}})$ thì $D=\frac{7\sqrt{3}}{6} > 2$
Vậy dự là $Min=2$
Ta CM:
$D=\frac{1}{a+b}+\frac{1}{b+c}+\frac{1}{c+a}-\frac{1}{a+b+c}\geq 2$
Nhân 2 vế $a+b+c$
$\Leftrightarrow \frac{a}{b+c}+\frac{b}{c+a}+\frac{c}{b+a}+2\geq 2(a+b+c)$
Nếu ta sử dụng BĐT $Nesbit$ thì $a=b=c$. Vậy ta sử dụng
$\frac{a}{b+c}+\frac{b}{c+a}+\frac{c}{a+b}\geq \frac{(a+b+c)^2}{2(ab+bc+ca)}=\frac{(a+b+c)^2}{2}$
Vậy ta cần CM :
$\frac{(a+b+c)^2}{2}+2\geq 2(a+b+c)$ ( đúng theo $AM-GM$)
Vậy ta có $ĐPCM$
(lưu ý : BĐT $Schwarz$ tử = 0 thì mẫu = 0)

Bài viết đã được chỉnh sửa nội dung bởi yeutoan11: 07-10-2012 - 18:49

Dựng nước lấy việc học làm đầu. Muốn thịnh trị lấy nhân tài làm gốc.
NGUYỄN HUỆ
Nguyễn Trần Huy
Tự hào là thành viên VMF

#62
tim1nuathatlac

tim1nuathatlac

    Thượng sĩ

  • Thành viên
  • 298 Bài viết

Bài 39.
Ch0 $x,y,z>0$ thỏa $xy+yz+zx=1$.Chứng minh rằng:
$$\frac{x}{y(1+x^2)}+\frac{y}{z(1+y^2)}+\frac{z}{x(1+z^2)}\geq \frac{9}{4}$$


Đề phải như vậy chứ.

$Q.e.D\Leftrightarrow \sum \frac{x}{y\left ( x+y \right )\left ( x+z \right )}\geq \frac{9}{4}$

$VT=\sum \frac{x^{2}z}{xyz\left ( x+y \right )\left ( x+z \right )}=\frac{\sum x^{2}z\left ( y+z \right )}{xyz\left ( x+y \right )\left ( x+z \right )\left ( y+z \right )}$

Ta có 1 số phép biến đổi sau

$\sum x^{2}z\left ( y+z \right )=\left ( xy+yz+zx \right )^{2}-2xyz\left ( x+y+z \right )$

$\left ( x+y+z \right )=\left ( x+y+z \right )\left ( xy+yz+zx \right )\leq \frac{9}{8}\prod \left ( x+y \right )$

$xyz\prod \left ( x+y \right )\leq \frac{8\left ( xy+yz+zx \right )^{3}}{27}=\frac{8}{27}$

Áp dụng vào bđt ta được:

$VT=\frac{\left ( xy+yz+zx \right )^{2}}{xyz\left ( x+y \right )\left ( y+z \right )\left ( z+x \right )}-\frac{x+y+z}{\left ( x+y \right )\left ( y+z \right )\left ( z+x \right )}$

$=\frac{27}{8}-\frac{9}{8}=\frac{9}{4}$

Vậy bđt được cm, Mình có bài dưới nhẹ hơn

Bài 42 Cho $x,y,z$ là các số thực dương thỏa $xy+yz+zx=1$.CMR $\frac{x}{y\left ( 1+z^{2} \right )}+\frac{y}{z\left ( 1+x^{2} \right )}+\frac{z}{x\left ( 1+y^{2} \right )}\geq \frac{9}{4}$


#63
lehoanghiep

lehoanghiep

    Trung sĩ

  • Thành viên
  • 196 Bài viết
Bài 43 Cho các số thực dương $x,y,z$ thỏa mãn $4\left ( x+y+z \right )=3xyz$. Tìm giá trị lớn nhất của biểu thức $P=\frac{1}{2+x+yz}+\frac{1}{2+y+zx}+\frac{1}{2+z+xy}$.

#64
lehoanghiep

lehoanghiep

    Trung sĩ

  • Thành viên
  • 196 Bài viết


Bài 8.[ Thi thử lần 2 THPT Trung Giã]
Cho các số thực dương $a,b,c$ thoả mãn $abc=1$. Tìm GTLN của :
$$P=\dfrac{1}{\sqrt{a}+\sqrt{b}+2\sqrt{c}+2}+\dfrac{1}{\sqrt{b}+\sqrt{c}+2\sqrt{a}+2}+\dfrac{1}{\sqrt{c}+\sqrt{a}+2\sqrt{b}+2}$$

Ta có $\sqrt{a}+\sqrt{c}+\sqrt{b}+\sqrt{c}\geq 2\sqrt[4]{ca}+2\sqrt[4]{bc}$.
Đặt $\sqrt[4]{ab}=x^{3};\sqrt[4]{bc}=y^{3};\sqrt[4]{ca}=z^{3}$.
Khi đó $2P\leq \frac{1}{x^{3}+y^{3}+1}+\frac{1}{y^{3}+z^{3}+1}+\frac{1}{z^{3}+x^{3}+1}$ với $xyz=1$.
Ta có BĐT $x^{3}+y^{3}\geq xy\left ( x+y \right )\Leftrightarrow \left ( x+y \right )\left ( x-y \right )^{2}\geq 0$ ($x,y>0$).
Áp dụng BĐT trên, ta được $2P\leq \frac{1}{\frac{x+y}{z}+1}+\frac{1}{\frac{y+z}{x}+1}+\frac{1}{\frac{z+x}{y}+1}=1$.
Vậy $P_{max}=\frac{1}{2}$. Đẳng thức xảy ra khi $a=b=c=1$.

#65
NGOCTIEN_A1_DQH

NGOCTIEN_A1_DQH

    Never Give Up

  • Thành viên
  • 625 Bài viết

Bài 43 Cho các số thực dương $x,y,z$ thỏa mãn $4\left ( x+y+z \right )=3xyz$. Tìm giá trị lớn nhất của biểu thức $P=\frac{1}{2+x+yz}+\frac{1}{2+y+zx}+\frac{1}{2+z+xy}$.

làm bài này :D :D

ta có: $ GT \Leftrightarrow \frac{4}{xy}+\frac{4}{yz}+\frac{4}{zx} = 3 $

đặt $ a=\frac{1}{x}; b=\frac{1}{y}; c=\frac{1}{z} $ thì ta có $ ab+bc+ca=\frac{3}{4} $ và:

$ P=\frac{abc}{2abc+bc+a}+\frac{abc}{2abc+ca+b}+\frac{abc}{2abc+ab+c} $

áp dụng bất đẳng thức AM-GM ta có:

$ 2abc+bc+a=2abc+bc+\frac{a}{2}+\frac{a}{2} \geq 4\sqrt[4]{\frac{a^3b^2c^2}{2}} $

$ \Rightarrow \frac{abc}{2abc+bc+a} \leq \frac{\sqrt[4]{2ab^2c^2}}{4} \leq \frac{\sqrt{2abc}+\sqrt{bc}}{8} \leq \frac{2abc+\frac{1}{4}+bc+\frac{1}{4}}{8} $

làm tương tự rồi cộng lại ta có:

$ P \leq \frac{6abc+ab+bc+ca+\frac{3}{2}}{8} \leq \frac{3}{8} $ (do $ ab+bc+ca=\frac{3}{4} $ và $ abc \leq \frac{1}{8} $)

vậy $ maxP=\frac{3}{8} \Leftrightarrow x=y=z=2 $

p/s: tính nhầm chút, đã sửa :icon6: :wub:

Bài viết đã được chỉnh sửa nội dung bởi NGOCTIEN_A1_DQH: 08-10-2012 - 20:30

Em cắm hoa tươi đặt cạnh bàn

Mong rằng toán học bớt khô khan

Em ơi trong toán nhiều công thức

Cũng đẹp như hoa lại chẳng tàn

#66
lehoanghiep

lehoanghiep

    Trung sĩ

  • Thành viên
  • 196 Bài viết

Bài 14.[ Đề thi thử THPT Chuyên Phan Bội Châu Nghê An]
Ch0 các số thực $a,b,c$ thỏa $0\leq a\leq b\leq c$ và $a^2+b^2+c^2=3$.Tìm GTLN của:
$$A=5a-4abc$$

Ai có lời giải cho bài này không?

#67
sogenlun

sogenlun

    Trung sĩ

  • Thành viên
  • 105 Bài viết

Bài 43 Cho các số thực dương $x,y,z$ thỏa mãn $4\left ( x+y+z \right )=3xyz$. Tìm giá trị lớn nhất của biểu thức $P=\frac{1}{2+x+yz}+\frac{1}{2+y+zx}+\frac{1}{2+z+xy}$.

Một cách tổng quát :Với $a,b \ge 1 , c \ge b , c \ge 2 , abc \in \mathbb{R}$
Trước hết : từ giả thiết ta có :
$$\begin{cases} \dfrac{1}{xy}+\dfrac{1}{yz}+\dfrac{1}{xz}= \dfrac{3}{4} \\ xyz \ge 8 \end{cases} $$
Và để ý một chút sẽ thấy :
$$\dfrac{1}{x+1}+\dfrac{1}{y+1}+\dfrac{1}{1+z} \le 1$$
Thật vậy , nhân tung hết rồi rút gọn nó tương đương với : $$xyz \ge x+y+z + 2$$
$$ \Leftrightarrow \dfrac{xyz}{4} \ge 2$$
Áp dụng vào đây ta có : Sử dụng $Cauchy-Schwarz$:
$$\dfrac{a}{yz}+\dfrac{b.(\dfrac{3}{4})^2}{1+x}+(c-b).\dfrac{(\dfrac{1}{4})^2}{1} = \dfrac{1}{yz}+...+\dfrac{1}{yz}+\dfrac{(\dfrac{3}{4})^2}{1+x}+...+\dfrac{(\dfrac{3}{4})^2}{1+x}+\dfrac{(\dfrac{1}{4})^2}{1}+..+\dfrac{(\dfrac{1}{4})^2}{1} \ge \dfrac{(a+b.\dfrac{3}{4}+(c-b).\dfrac{1}{4})^2}{ayz+bx+c} $$
Thiết lập các BĐT tương tự rồi cộng vế ta được :
$$ a(\dfrac{1}{xy}+\dfrac{1}{yz}+\dfrac{1}{xz})+ \dfrac{9b}{16}.(\dfrac{1}{x+1}+\dfrac{1}{y+1}+\dfrac{1}{1+z}) + 3(c-b).\dfrac{1}{16} \ge \sum \dfrac{(a+\dfrac{b}{2}+\dfrac{c}{4})^2}{ayz+bx+c} $$
Suy ra $$\sum \dfrac{1}{ayz+bx+c} \le \dfrac{3}{4a+2b+c} $$
Và đẳng thức đạt được tại $x=y=z =2$ :)

Bài viết đã được chỉnh sửa nội dung bởi sogenlun: 09-10-2012 - 12:52

Chia sẻ tài liệu ôn thi đại học tại : http://blogtoanli.net


#68
NGOCTIEN_A1_DQH

NGOCTIEN_A1_DQH

    Never Give Up

  • Thành viên
  • 625 Bài viết
bài 44: cho $ x, y, z \in \mathbb{R} $ thỏa $ 3^{-x}+3^{-y}+3^{-z}=1$. chứng minh rằng:

$$ \frac{9^x}{3^x+3^{y+z}}+\frac{9^y}{3^y+3^{z+x}}+\frac{9^z}{3^z+3^{x+y}} \geq \frac{3^x+3^y+3^z}{4} $$

đề thi thử THPT Dương Quảng Hàm 2008-2009


Bài viết đã được chỉnh sửa nội dung bởi NGOCTIEN_A1_DQH: 09-10-2012 - 19:50

Em cắm hoa tươi đặt cạnh bàn

Mong rằng toán học bớt khô khan

Em ơi trong toán nhiều công thức

Cũng đẹp như hoa lại chẳng tàn

#69
Tham Lang

Tham Lang

    Thượng úy

  • Thành viên
  • 1149 Bài viết

Bài 24 ( ĐH khối A năm 2009 )
CMR vs mọi số thực dương x,y,z thỏa mãn $x\left ( x+y+z \right )=3yz$ ta cố

$\left ( x+y \right )^{3}+\left ( x+z \right )^{3}+3\left ( x+y \right )\left ( y+z \right )\left ( z+x \right )\leq 5\left ( y+z \right )^{3}$

BÀI TOÁN. Chứng minh rằng với mọi số thực dương $x,y,z$ thỏa mãn $x(x+y+z)=3yz$, ta có:
$$(x+y)^{3}+(x+z)^{3}+3(x+y)(x+z)(y+z)\leq 5(y+z)^{3}$$
GIẢI.

Cách 1 [HÀ QUỐC ĐẠT]

Đặt $a=x+y;b=x+z;c=y+z \Rightarrow x=\dfrac{a+b-c}{2};y=\dfrac{a+c-b}{2};z=\dfrac{b+c-a}{2}$
Khi đó: $$x(x+y+z)=3yz = \dfrac{a+b-c}{2}.\dfrac{a+b+c}{2}=3\dfrac{a+c-b}{2}.\dfrac{b+c-a}{2}$$
$$\Leftrightarrow (a+b)^{2}-c^{2}=3[c^{2}-(b-a)^{2}]$$
$$\Leftrightarrow a^{2}+b^{2}-c^{2}+2ab=-3a^{2}-3b^{2}+3c^{2}+6ab$$
$$\Leftrightarrow 4c^{2}=4a^{2}+4b^{2}-4ab$$
$$\Leftrightarrow c^{2}=a^{2}+b^{2}-ab\geq \dfrac{(a+b)^{2}}{4}\geq ab \Leftrightarrow 2c\geq a+b$$
Ta có: $3c^{3}\geq 3abc$
$$\Leftrightarrow 2c^{3}-(a^{3}+b^{3})=2c^{3}-(a+b)(a^{2}-ab+b^{2})=2c^{3}-(a+b)c^{2}=c^{2}[2c-(a+b)]\geq 0$$
$$\Leftrightarrow 2c^{3}\geq a^{3}+b^{3} \Leftrightarrow 5c^{3}\geq a^{3}+b^{3}+3abc$$
$$\Leftrightarrow 5(y+z)^{3}\geq (x+y)^{3}+(y+z)^{3}+3(x+y)(y+z)(z+x)$$
Dẫu "=" xảy ra khi $x=y=z=1$

Cách 2 [WWW]
Đặt: $y = ax;z = bx$. Khi đó $(1)$ trở thành: $x\left( {x + ax + bx} \right) = 3ab{x^2} \Leftrightarrow 1 + a + b = 3ab\,\,\,(3)$

và $(2)$ trở thành: ${\left( {x + ax} \right)^3} + {\left( {x + bx} \right)^3} + 3\left( {x + ax} \right)\left( {ax + bx} \right)\left( {bx + x} \right) \le 5{\left( {ax + bx} \right)^3}$
$$ \Leftrightarrow {\left( {1 + a} \right)^3} + {\left( {1 + b} \right)^3} + 3\left( {1 + a} \right)\left( {1 + b} \right)\left( {a + b} \right) \le 5{\left( {a + b} \right)^3}\,\,\,(4)$$
Vì $(3)$ và $(4)$ là những biểu thức đối xứng với $a, b$ nên đặt $S = a + b;\,P = ab$, khi đó
$$\left\{ \begin{array}{l}{S^2} \ge 4P\\1 + S = 3P\end{array} \right. \Leftrightarrow \left\{ \begin{array}{l}P = \dfrac{{1 + S}}{3}\\3{S^2} - 4S - 4 \ge 0\end{array} \right. \Leftrightarrow \left\{ \begin{array}{l}P = \dfrac{{1 + S}}{3}\\S \ge 2\end{array} \right.$$
Suy ra $$\left( {1 + a} \right)\left( {1 + b} \right) = 1 + a + b + ab = 1 + S + \dfrac{{1 + S}}{3} = \dfrac{{4\left( {1 + S} \right)}}{3}$$
$${\left( {1 + a} \right)^3} + {\left( {1 + b} \right)^3} = {\left( {2 + a + b} \right)^3} - 3\left( {1 + a} \right)\left( {1 + b} \right)\left( {2 + a + b} \right) = {\left( {2 + S} \right)^3} - 4\left( {1 + S} \right)\left( {2 + S} \right)$$
$$\left( 4 \right) \Leftrightarrow {\left( {2 + S} \right)^3} - 4\left( {{S^2} + 3S + 2} \right) + 4S\left( {1 + S} \right) \le 5{S^3}$$
$$ \Leftrightarrow 2{S^2} - 3S - 2 \ge 0 \Leftrightarrow \left( {2S + 1} \right)\left( {S - 2} \right) \ge 0\,\,\,\text{ luôn đúng do}\,\,S \ge 2$$
Vậy ta có đpcm.

Cách 3: [WWW]
$\left( 1 \right) \Leftrightarrow {x^2} + xy + x{\rm{z}} = 3yz \Rightarrow {(x + y)^2} + {(x + z)^2} = 2{(y + z)^2} - {(y - z)^2}$

$ \Rightarrow {\left( {\dfrac{{x + y}}{{y + z}}} \right)^2} + {\left( {\dfrac{{x + z}}{{y + z}}} \right)^2} = 2 - {\left( {\dfrac{{x + y}}{{y + z}} - \dfrac{{x + z}}{{y + z}}} \right)^2}\,\,\,\,\left( 3 \right)$

Đặt $u = \dfrac{{x + y}}{{y + z}},\,\,v = \dfrac{{x + z}}{{y + z}}$. Từ (3) $ \Rightarrow {u^2} + {v^2} = 2 - {(u - v)^2} \Rightarrow {u^2} + {v^2} - uv = 1\,\,\,\,\,\left( 4 \right)$

Khi đó ta có: BĐT $ \Leftrightarrow {\left( {\dfrac{{x + y}}{{y + z}}} \right)^3} + {\left( {\dfrac{{x + z}}{{y + z}}} \right)^3} + 3\left( {\dfrac{{x + y}}{{y + z}}} \right)\left( {\dfrac{{x + z}}{{y + z}}} \right) \le 5 \Leftrightarrow {u^3} + {v^3} + 3uv \le 5$

$ \Leftrightarrow (u + v)({u^2} - uv + {v^2}) + 3uv \le 5 \Leftrightarrow u + v + 3uv \le 5\,\,\,\left( 5 \right)$

Mặt khác từ (1) ta có: $uv = 1 - {(u - v)^2} \le 1\,\,\,\left( 6 \right)$

và ${(u + v)^2} = 1 + 3uv \le 1 + \dfrac{3}{4}{(u + v)^2} \Rightarrow {(u + v)^2} \le 4 \Rightarrow u + v \le 2\,\,\,\left( 7 \right)$

Từ $(6)$ và $(7)$ ta có đpcm.

Bài 32 (Thi thử THPT Chuyên ĐH Vinh lần 2-2012)
Cho các số thực $a,b,c$ thuộc đoạn $\left [ 0;1 \right ]$. Tìm giá trị lớn nhất của biểu thức $P=\frac{a^{3}+2}{b^{2}+1}+\frac{b^{3}+2}{c^{2}+1}+\frac{c^{3}+2}{a^{2}+1}$.

$P\le \dfrac{a^2+2}{b^2+1}+\dfrac{b^2+2}{c^2+1}+\dfrac{c^2+2}{a^2+1}$
Chứng minh VP $\le 6$ bằng cách biến đổi tương đương.

Off vĩnh viễn ! Không ngày trở lại.......


#70
Tham Lang

Tham Lang

    Thượng úy

  • Thành viên
  • 1149 Bài viết

bài 44: cho $ x, y, z \in \mathbb{R} $ thỏa $ 3^{-x}+3^{-y}+3^{-z}=1$. chứng minh rằng:

$$ \frac{9^x}{3^x+3^{y+z}}+\frac{9^y}{3^y+3^{z+x}}+\frac{9^z}{3^z+3^{x+y}} \geq \frac{3^x+3^y+3^z}{4} $$

đề thi thử THPT Dương Quảng Hàm 2008-2009

Đặt $3^x=a, 3^y=b, 3^z=c$ thì $ab+bc+ca=abc$
Ta cần chứng minh :
$$\dfrac{a^2}{a+bc}+\dfrac{b^2}{b+ca}+\dfrac{c^2}{c+ab}\ge \dfrac{a+b+c}{4}$$
Theo CS, ta có :
$$\dfrac{a^2}{a+bc}+\dfrac{b^2}{b+ca}+\dfrac{c^2}{c+ab}=\dfrac{a^3}{a^2+abc}+\dfrac{b^3}{b^2+abc}+\dfrac{c^3}{c^2+abc} \ge \dfrac{(a+b+c)^3}{3\left (a^2+b^2+c^2+3abc\right )} =\dfrac{(a+b+c)^3}{3\left [(a+b+c)^2+ab+bc+ca\right ]} \ge \dfrac{(a+b+c)^3}{4(a+b+c)^2} =\dfrac{a+b+c}{4}$$
Đẳng thức xảy ra khi $a=b=c \Leftrightarrow x=y=z=1$

Off vĩnh viễn ! Không ngày trở lại.......


#71
Tham Lang

Tham Lang

    Thượng úy

  • Thành viên
  • 1149 Bài viết

Bài 43 Cho các số thực dương $x,y,z$ thỏa mãn $4\left ( x+y+z \right )=3xyz$. Tìm giá trị lớn nhất của biểu thức $P=\frac{1}{2+x+yz}+\frac{1}{2+y+zx}+\frac{1}{2+z+xy}$.

Một hướng khác cho bài toán này, nhưng phải sử dụng AM-GM cho 3, 4 số..
Ta có :
$$\dfrac{1}{2+x+yz} \le \dfrac{1}{4\dfrac{\sqrt[4]{xy^2z^2}}{2}}=\dfrac{1}{\dfrac{4}{\sqrt[4]{2}}.\sqrt[8]{x^2y^4z^4}}$$
Đặt $\sqrt[8]{yz}=a, \sqrt[8]{zx}=b, \sqrt[8]{xy}=c$ thì ta có:
$$\dfrac{1}{a^8}+\dfrac{1}{b^8}+\dfrac{1}{c^8} =\dfrac{3}{4}$$
Cần chứng minh :
$$\dfrac{\sqrt[4]{2}}{4abc}\left (\dfrac{1}{a^2}+\dfrac{1}{b^2}+\dfrac{1}{c^2}\right ) \le \dfrac{3}{8}$$
Đây là một BĐT rất lỏng, không khó để chứng minh.

Off vĩnh viễn ! Không ngày trở lại.......


#72
sogenlun

sogenlun

    Trung sĩ

  • Thành viên
  • 105 Bài viết

Đặt $3^x=a, 3^y=b, 3^z=c$ thì $ab+bc+ca=abc$
Ta cần chứng minh :
Theo CS, ta có :
$$\dfrac{a^3}{a^2+abc}+\dfrac{b^3}{b^2+abc}+\dfrac{c^3}{c^2+abc} \ge \dfrac{(a+b+c)^3}{3\left (a^2+b^2+c^2+3abc\right )}$$

Đoạn này mình thấy không ổn :D

Bài viết đã được chỉnh sửa nội dung bởi sogenlun: 10-10-2012 - 06:26

Chia sẻ tài liệu ôn thi đại học tại : http://blogtoanli.net


#73
Tham Lang

Tham Lang

    Thượng úy

  • Thành viên
  • 1149 Bài viết

Đoạn này mình thấy không ổn :D

Nói đúng hơn là CS suy rộng, hay Holder cũng được .
$$(1+1+1)(a^2+abc+b^2+abc+c^2+abc).VT \ge (a+b+c)^3$$

Off vĩnh viễn ! Không ngày trở lại.......


#74
sogenlun

sogenlun

    Trung sĩ

  • Thành viên
  • 105 Bài viết

Nói đúng hơn là CS suy rộng, hay Holder cũng được .
$$(1+1+1)(a^2+abc+b^2+abc+c^2+abc).VT \ge (a+b+c)^3$$

Vì là ôn thi đại học nên mình nghĩ không xài $Holder$.
Mình xin giải như sau :
Sau khi đổi biến thành chứng minh
$$\dfrac{a^2}{a+bc}+\dfrac{b^2}{b+ca}+\dfrac{c^2}{c+ab}\ge \dfrac{a+b+c}{4}$$
với $ab+bc+ca=abc$.
Ta có :
$$\dfrac{a^2}{a+bc} = \dfrac{a^2.abc}{a.abc+bc.(ab+bc+ca)} = \dfrac{a^3bc}{(bc+ab)(bc+ac)} = \dfrac{a^3}{(a+b)(a+c)}$$
Thiết lập các đẳng tương tự , ta đưa về chứng minh :
$$\dfrac{a^3}{(a+b)(a+c)}+\dfrac{b^3}{(a+b)(b+c)}+\dfrac{c^3}{(a+c)(c+b)} \ge \dfrac{a+b+c}{4}$$
Thật vậy , áp dụng BĐT $AM-GM$ có :
$$\dfrac{a^3}{(a+b)(a+c)} +\dfrac{a+b}{8}+\dfrac{a+c}{8} \ge \dfrac{3a}{4}$$
$$ \Rightarrow \dfrac{a^3}{(a+b)(a+c)} \ge \dfrac{a}{2}-\dfrac{b+c}{8}$$
Thiết lập các BĐT tương tự , ta có $$VT \ge \dfrac{a+b+c}{2}- \dfrac{a+b+c}{4} = \dfrac{a+b+c}{4}$$
Đpcm.
Đẳng thức xảy ra khi $a=b=c=3$ hay $x=y=z=1$ :)

Chia sẻ tài liệu ôn thi đại học tại : http://blogtoanli.net


#75
tim1nuathatlac

tim1nuathatlac

    Thượng sĩ

  • Thành viên
  • 298 Bài viết
Bài 45 Cho $a,b,c$ là các số thực dương tùy ý. CM $\frac{ab}{c^{2}}+\frac{bc}{a^{2}}+\frac{ca}{b^{2}}\geq \frac{1}{2}\left ( \frac{a+b}{c}+\frac{b+c}{a}+\frac{c+a}{b} \right )$

Bài 46 Cho $a,b,c$ là các số thực dương, thỏa mãn $a^{7}+b^{7}+c^{7}\leq 3$. CM $\frac{1}{2-a}+\frac{1}{2-b}+\frac{1}{2-c}\leq 3$
Gợi ý CM $\frac{1}{2-a}\leq \frac{a^{7}+6}{7}$

Bài 47 Cho $a,b,c$ là các số thực không âm, thỏa mãn $ab+bc+ca> 0$.

CM $\frac{a}{b^{3}+c^{3}}+\frac{b}{c^{3}+a^{3}}+\frac{c}{a^{3}+b^{3}}\geq \frac{8}{\left ( a+b+c \right )^{2}}$

Bài 48 Cho $a,b,c$ là các số không âm. CM $\left ( a^{2}+b^{2}+c^{2} \right )^{2}\geq 4\left ( a+b+c \right )\left ( a-b \right )\left ( b-c \right )\left ( c-a \right )$

Gợi ý Gs $\left ( b-a \right )\left ( b-c \right )\leq 0$
Vs $a\geq b\geq c$ thì $VT\geq 0\geq VP$
Vs $c\geq b\geq a\Rightarrow VP\leq \left [ \left ( c-a \right )\left ( c-b \right )+\left ( b-a \right )\left ( a+b+c \right ) \right ]^{2}$

Ctrl A

Bài viết đã được chỉnh sửa nội dung bởi tim1nuathatlac: 11-10-2012 - 22:26
chỉnh sửa số bài



#76
lehoanghiep

lehoanghiep

    Trung sĩ

  • Thành viên
  • 196 Bài viết
Bài 49 Cho $a,b,c$ là các số thực dương thỏa mãn $a+b+c=\frac{1}{2}$. Tìn giá trị lớn nhất của biểu thức $P=\frac{ab}{\left ( 1-a \right )\left ( 1-b \right )}+\frac{bc}{\left ( 1-b \right )\left ( 1-c \right )}+\frac{ca}{\left ( 1-c \right )\left ( 1-a \right )}$.

#77
quoctruong1202

quoctruong1202

    Trung sĩ

  • Thành viên
  • 129 Bài viết
Bài 50:Xét các số thực x,y,z thỏa mãn x+y+z=0. Tìm Min

Q= $\left | 2x-y \right |+\left | 2y-x \right |+\left | 2z-x \right |-ln\left ( \sqrt{14\left ( x^{2}+y^{2}+z^{2} \right )} +1\right )$

Bài viết đã được chỉnh sửa nội dung bởi quoctruong1202: 15-10-2012 - 17:04

Hình đã gửi

#78
quoctruong1202

quoctruong1202

    Trung sĩ

  • Thành viên
  • 129 Bài viết
Bạn ơi gửi thêm bài về sử dụng phương pháp hàm số đi! Loại này thi đại học mấy năm nay hay dùng quá!
Hình đã gửi

#79
HÀ QUỐC ĐẠT

HÀ QUỐC ĐẠT

    Thượng sĩ

  • Thành viên
  • 295 Bài viết
Bài 45
quy đồng ta được
$$2(a^{3}b^{3}+b^{3}c^{3}+c^{3}a^{3})\geq abc[a^{2}(b+c)+b^{2}(c+a)+c^{2}(a+b)]$$
Áp dụng AM-GM ta được
$$2(a^{3}b^{3}+b^{3}c^{3}+c^{3}a^{3})\geq a^{3}b^{3}+b^{3}c^{3}+c^{3}a^{3}+3a^{2}b^{2}c^{2}$$
Như vậy ta cần chứng minh
$$a^{3}b^{3}+b^{3}c^{3}+c^{3}a^{3}+3a^{2}b^{2}c^{2}\geq abc[a^{2}(b+c)+b^{2}(c+a)+c^{2}(a+b)]$$
Đặt $x=ab;y=bc;z=ca$.Bất đẳng thức trên trở thành
$$x^{3}+y^{3}+z^{3}+3xyz\geq x^{2}(y+z)+y^{2}(z+x)+z^{2}(x+y)$$
Đây chính là bất đẳng thức Schur
Vậy ta có điều phải chứng minh

#80
tim1nuathatlac

tim1nuathatlac

    Thượng sĩ

  • Thành viên
  • 298 Bài viết
Bài 45 Cho $a,b,c$ là các số thực dương tùy ý. CM $\frac{ab}{c^{2}}+\frac{bc}{a^{2}}+\frac{ca}{b^{2}}\geq \frac{1}{2}\left ( \frac{a+b}{c}+\frac{b+c}{a}+\frac{c+a}{b} \right )$

Bài 47 Cho $a,b,c$ là các số thực không âm, thỏa mãn $ab+bc+ca> 0$.

CM $\frac{a}{b^{3}+c^{3}}+\frac{b}{c^{3}+a^{3}}+\frac{c}{a^{3}+b^{3}}\geq \frac{8}{\left ( a+b+c \right )^{2}}$

Bài 45 Chỉ 1 dòng thôi! :P Bất đẳng thức cần cm tương đương với $\sum \frac{a\left ( b+c \right )\left ( b-c \right )^{2}}{b^{2}c^{2}}\geq 0$

Bài 47 Sử dụng $Cauchy-schwarz$ ta có

$\sum \frac{a}{b^{3}+c^{3}}\geq \frac{\left ( a+b+c \right )^{2}}{\sum ab\left ( a^{2}+b^{2} \right )}\geq \frac{\left ( a+b+c \right )^{2}}{\sum ab\left ( a^{2}+b^{2}+c^{2} \right )}$

$\Leftrightarrow VT\geq \frac{\left ( a+b+c \right )^{2}}{\left ( a^{2} +b^{2}+c^{2} \right )\left ( ab+bc+ca \right )}\geq \frac{8}{\left ( a+b+c \right )^{2}}$


Chứng minh hoàn tất. Dấu = xảy ra khi $a=b,c=0$ vá các hoán vị của chúng. $\square$

Bài viết đã được chỉnh sửa nội dung bởi tim1nuathatlac: 15-10-2012 - 23:26






0 người đang xem chủ đề

0 thành viên, 0 khách, 0 thành viên ẩn danh